หัวข้อ: USAMO 2005
ดูหนึ่งข้อความ
  #6  
Old 16 มีนาคม 2007, 19:45
warut warut ไม่อยู่ในระบบ
กระบี่ไร้สภาพ
 
วันที่สมัครสมาชิก: 24 พฤศจิกายน 2001
ข้อความ: 1,627
warut is on a distinguished road
Smile

อ้างอิง:
ข้อความเดิมของคุณ gools:
2.Prove that the system
\[\begin{array}{rcl}
x^6+x^3+x^3y+y & = & 147^{157} \\
x^3+x^3y+y^2+y+z^9 & = & 157^{147}
\end{array}
\]
has no solutions in integers \(x\), \(y\), and \(z\).
ผมลองทำตาม hint ที่คุณ nongtum ให้มาคือใช้ modulo 13 ดังนั้นสมภาคทั้งหมดข้างล่างนี้เป็น mod 13 นะครับ นั่นทำให้สมการทั้งสองกลายเป็น $$\begin{array}{lrcl} (1) & x^6+x^3+x^3y+y & \equiv & 4 \\ (2) & x^3+x^3y+y^2+y+z^9 & \equiv & 1 \end{array}$$ เนื่องจาก $x^3\equiv 0, \pm1, \pm5$ และ $x^6\equiv 0, \pm1$ ดังนั้น $z^9= z^6z^3 \equiv 0, \pm1, \pm5$

กรณีที่ 1: $x^3\equiv0$
จาก $(1)$ เราจะได้ $y\equiv4$ แทนค่าลงใน $(2)$ จะได้ $z^9\equiv-6$ ดังนั้นจึงไม่มีคำตอบในกรณีนี้

กรณีที่ 2: $x^3\equiv1$
จาก $(1)$ เราจะได้ $y\equiv1$ แทนค่าลงใน $(2)$ จะได้ $z^9\equiv-3$ ดังนั้นจึงไม่มีคำตอบในกรณีนี้

กรณีที่ 3: $x^3\equiv-1$
เราจะพบว่าไม่มี $y$ ที่สอดคล้องกับ $(1)$ จึงไม่มีคำตอบในกรณีนี้

กรณีที่ 4: $x^3\equiv5$
จาก $(1)$ เราจะได้ $y\equiv0$ แทนค่าลงใน $(2)$ จะได้ $z^9\equiv-4$ ดังนั้นจึงไม่มีคำตอบในกรณีนี้

กรณีที่ 5: $x^3\equiv-5$
จาก $(1)$ เราจะได้ $y\equiv4$ แทนค่าลงใน $(2)$ จะได้ $z^9\equiv6$ ดังนั้นจึงไม่มีคำตอบในกรณีนี้เช่นกัน

เราจึงสรุปได้ว่า ระบบสมการของโจทย์ข้อนี้ ไม่มีคำตอบเป็นจำนวนเต็มครับ

ป.ล. ผมไม่ได้ลอง search หาเฉลยของข้อนี้นะครับ แต่คิดว่าน่าจะคล้ายกับที่ผมทำ เพราะส่วนที่ยากที่สุดน่าจะเป็นการหา modulus ที่เหมาะสม ซึ่งผมลองให้คอมพ์หา prime moduli ที่ใช้ได้ ก็เจอเพียง 13 กับ 19 ตามที่คุณ nongtum บอกไว้ใน hint นั่นแหละครับ
ตอบพร้อมอ้างอิงข้อความนี้